LSAT and Law School Admissions Forum

Get expert LSAT preparation and law school admissions advice from PowerScore Test Preparation.

 Administrator
PowerScore Staff
  • PowerScore Staff
  • Posts: 8917
  • Joined: Feb 02, 2011
|
#23656
Complete Question Explanation

Strengthen. The correct answer choice is (B)

In this stimulus the author presents a cause and effect argument, but, somewhat unusually for the LSAT, there are multiple causes allowed. We are asked to strengthen the desired outcome, that is to reduce fatalities, based on the causal relationships presented in the stimulus. We are told that many of the fatalities are caused by a cabin design, and then given the solution to this problem, removing the extra seats. The question asks us what other act would futher strengthen this conclusion, that removing the seats will decrease fatalities. The first sentence is the key; the collisions are increasing because the flights are increasing. Yes, many of the fatalities are caused by cabin design, but not all of them. The collisions themselves obviously cause some of them.


Answer Choice (A): This answer choice does not add anything to the conclusion. If they are already required to remove all of these seats, why does it matter if they buy the plane with the seats in them or not? This would be the answer if the conclusion said "remove all seats in existing planes' because then it would leave out new planes. The conclusion is not restricted in that way, though, so this is not the correct answer.

Answer Choice (B): This is the correct answer choice. It takes into account that first line of the stimulus. If the airlines increase the number of flights to make up for lost seats, then the collisions will increase and fatalities may increase as well. If the number of flights remain the same, however, removing the seats should reduce fatalities according to the premises.

Answer Choice (C): The answer choice is incorrect even though most of us would really like to see it happen. It has nothing to do with the stimulus and does not strengthen it in any way.

Answer Choice (D): This answer choice, like the previous one, is entirely unrelated to the issue in the stimulus: removing seats and its impact on fatalities. Although "security procedures" sound related, this answer choice does not strengthen the issue at all.

Answer Choice (E): This answer choice is incorrect, even though, again, we would like it to be upheld. Ticket prices have no affect on fatalities or collisions based on the information in the stimulus.
 GLMDYP
  • Posts: 100
  • Joined: Aug 19, 2013
|
#12458
Hi!
I chose (A) but he answer is (B). I recognized that (B) is the right answer, but I think (A) is still a very good choice. If we don't block out the doorway, there will be less fatalities. Isn't it so? I noticed that there're two reasons simultaneously responsible for the fatality rates though.
Thanks!
 David Boyle
PowerScore Staff
  • PowerScore Staff
  • Posts: 836
  • Joined: Jun 07, 2013
|
#12477
GLMDYP wrote:Hi!
I chose (A) but he answer is (B). I recognized that (B) is the right answer, but I think (A) is still a very good choice. If we don't block out the doorway, there will be less fatalities. Isn't it so? I noticed that there're two reasons simultaneously responsible for the fatality rates though.
Thanks!
Hello,

If this is the question beginning, "The number of aircraft collisions on the ground is increasing because of the substantial increase in the number of flights operated by the airlines", then while A is not a terrible choice: still, airlines don't have to BUY planes with clear access, they could just buy other planes and then rearrange the seats.

David
 GLMDYP
  • Posts: 100
  • Joined: Aug 19, 2013
|
#12538
Oh I get it! Thanks!
 lathlee
  • Posts: 652
  • Joined: Apr 01, 2016
|
#42065
Hi. Even though I do not argue that B is not incorrect (B is correct answer ) ; Isn't this Strengthen-principle instead of Strengthen, due to the phrase of Which one of the following proposals?
 nicholaspavic
PowerScore Staff
  • PowerScore Staff
  • Posts: 271
  • Joined: Jun 12, 2017
|
#42335
Hi lathlee,

You raise an interesting question, and there definitely are propositions in the answer choices. Of course Principle questions are not their own "type" of question, rather we consider them overlays, sometimes narrow and sometimes broad, often conditional and opinion-based. I'm inclined to agree with our original assessment here that the stimulus (which is what we initially assess is concerned with causality) though certainly the last sentence is giving is a principle (or proposition) that is strengthening the causality. That's at the heart of the correct answer too. Namely, strengthening that first sentence in a causal way. But I think you also have a valid point about this unusual question. Because there is some principle in the answer choices too! So well done and keep up the good work. :-D

Get the most out of your LSAT Prep Plus subscription.

Analyze and track your performance with our Testing and Analytics Package.